IMO 1994 Q1 #
THIS FILE IS SYNCHRONIZED WITH MATHLIB4. Any changes to this file require a corresponding PR to mathlib4.
Let m
and n
be two positive integers.
Let a₁, a₂, ..., aₘ
be m
different numbers from the set {1, 2, ..., n}
such that for any two indices i
and j
with 1 ≤ i ≤ j ≤ m
and aᵢ + aⱼ ≤ n
,
there exists an index k
such that aᵢ + aⱼ = aₖ
.
Show that (a₁+a₂+...+aₘ)/m ≥ (n+1)/2
Sketch of solution #
We can order the numbers so that a₁ ≤ a₂ ≤ ... ≤ aₘ
.
The key idea is to pair the numbers in the sum and show that aᵢ + aₘ₊₁₋ᵢ ≥ n+1
.
Indeed, if we had aᵢ + aₘ₊₁₋ᵢ ≤ n
, then a₁ + aₘ₊₁₋ᵢ, a₂ + aₘ₊₁₋ᵢ, ..., aᵢ + aₘ₊₁₋ᵢ
would be m
elements of the set of aᵢ
's all larger than aₘ₊₁₋ᵢ
, which is impossible.